Is it possible to find matrices $M, N$ such that $MXN$ has specified structure for all $X$?











up vote
1
down vote

favorite












Suppose $M in mathbb R^{2 times *}$ and $N in mathbb R^{# times 2}$. Here $*$ and $#$ means we can pick any dimension we want. The end goal is to find fixed $M, N$ such that range of the linear transformation $T: mathbb R^{* times #} to mathbb R^{2 times 2}$ given by $X mapsto MXN$ is the $1$-dim space spanned by $pmatrix{0 & 1 \ 1 & 0}$. I tried to pick $* = 1, 2$ and $#= 1, 2$ but with no luck. Is this even possible?










share|cite|improve this question
























  • What if $X=0$ and $alpha neq 0$?
    – parsiad
    Nov 22 at 3:29










  • @parsiad: Sorry, this is unclear formulation. I really mean: can we find fixed $M, N$ such that range of linear transformation $T: mathbb R^{* times #} to mathbb R^{2 times 2}$ given by $X mapsto MXN$ is the $1$-dim space spanned by $pmatrix{0 & 1 \ 1 & 0}$.
    – user1101010
    Nov 22 at 3:40

















up vote
1
down vote

favorite












Suppose $M in mathbb R^{2 times *}$ and $N in mathbb R^{# times 2}$. Here $*$ and $#$ means we can pick any dimension we want. The end goal is to find fixed $M, N$ such that range of the linear transformation $T: mathbb R^{* times #} to mathbb R^{2 times 2}$ given by $X mapsto MXN$ is the $1$-dim space spanned by $pmatrix{0 & 1 \ 1 & 0}$. I tried to pick $* = 1, 2$ and $#= 1, 2$ but with no luck. Is this even possible?










share|cite|improve this question
























  • What if $X=0$ and $alpha neq 0$?
    – parsiad
    Nov 22 at 3:29










  • @parsiad: Sorry, this is unclear formulation. I really mean: can we find fixed $M, N$ such that range of linear transformation $T: mathbb R^{* times #} to mathbb R^{2 times 2}$ given by $X mapsto MXN$ is the $1$-dim space spanned by $pmatrix{0 & 1 \ 1 & 0}$.
    – user1101010
    Nov 22 at 3:40















up vote
1
down vote

favorite









up vote
1
down vote

favorite











Suppose $M in mathbb R^{2 times *}$ and $N in mathbb R^{# times 2}$. Here $*$ and $#$ means we can pick any dimension we want. The end goal is to find fixed $M, N$ such that range of the linear transformation $T: mathbb R^{* times #} to mathbb R^{2 times 2}$ given by $X mapsto MXN$ is the $1$-dim space spanned by $pmatrix{0 & 1 \ 1 & 0}$. I tried to pick $* = 1, 2$ and $#= 1, 2$ but with no luck. Is this even possible?










share|cite|improve this question















Suppose $M in mathbb R^{2 times *}$ and $N in mathbb R^{# times 2}$. Here $*$ and $#$ means we can pick any dimension we want. The end goal is to find fixed $M, N$ such that range of the linear transformation $T: mathbb R^{* times #} to mathbb R^{2 times 2}$ given by $X mapsto MXN$ is the $1$-dim space spanned by $pmatrix{0 & 1 \ 1 & 0}$. I tried to pick $* = 1, 2$ and $#= 1, 2$ but with no luck. Is this even possible?







linear-algebra matrices






share|cite|improve this question















share|cite|improve this question













share|cite|improve this question




share|cite|improve this question








edited Nov 22 at 3:41

























asked Nov 22 at 3:26









user1101010

6341630




6341630












  • What if $X=0$ and $alpha neq 0$?
    – parsiad
    Nov 22 at 3:29










  • @parsiad: Sorry, this is unclear formulation. I really mean: can we find fixed $M, N$ such that range of linear transformation $T: mathbb R^{* times #} to mathbb R^{2 times 2}$ given by $X mapsto MXN$ is the $1$-dim space spanned by $pmatrix{0 & 1 \ 1 & 0}$.
    – user1101010
    Nov 22 at 3:40




















  • What if $X=0$ and $alpha neq 0$?
    – parsiad
    Nov 22 at 3:29










  • @parsiad: Sorry, this is unclear formulation. I really mean: can we find fixed $M, N$ such that range of linear transformation $T: mathbb R^{* times #} to mathbb R^{2 times 2}$ given by $X mapsto MXN$ is the $1$-dim space spanned by $pmatrix{0 & 1 \ 1 & 0}$.
    – user1101010
    Nov 22 at 3:40


















What if $X=0$ and $alpha neq 0$?
– parsiad
Nov 22 at 3:29




What if $X=0$ and $alpha neq 0$?
– parsiad
Nov 22 at 3:29












@parsiad: Sorry, this is unclear formulation. I really mean: can we find fixed $M, N$ such that range of linear transformation $T: mathbb R^{* times #} to mathbb R^{2 times 2}$ given by $X mapsto MXN$ is the $1$-dim space spanned by $pmatrix{0 & 1 \ 1 & 0}$.
– user1101010
Nov 22 at 3:40






@parsiad: Sorry, this is unclear formulation. I really mean: can we find fixed $M, N$ such that range of linear transformation $T: mathbb R^{* times #} to mathbb R^{2 times 2}$ given by $X mapsto MXN$ is the $1$-dim space spanned by $pmatrix{0 & 1 \ 1 & 0}$.
– user1101010
Nov 22 at 3:40












1 Answer
1






active

oldest

votes

















up vote
1
down vote



accepted










No, it's not possible unless you accept that the range is ${0}$. Consider first the case $1,1$. We have
$$
begin{bmatrix} M_{11}\ M_{21}end{bmatrix}begin{bmatrix} N_{11}&N_{12}end{bmatrix}=begin{bmatrix} 0&a\a&0end{bmatrix}.
$$

Then $M_{11}N_{11}=0$. If $M_{11}ne0$, then $N_{11}=0$. But then $a=M_{21}N_{11}=0$. From the $1,2$ entry, $0=a=M_{11}N_{12}$, $N_{12}=0$ and $N=0$. When $N_{11}=0$, we obtain similarly that $M=0$ (just take transpose and apply the argument).



Now in general, if $E_{kj}$ is the matrix with $1$ in the $k,j$ entry and zeroes elsewhere,
$$
ME_{kj}N=M_kN_j,
$$

where $M_k$ is the $k^{rm th}$ column of $M$ and $N_j$ with the $j^{rm th}$ row of $j$. By the previous case, one of them has to be zero. So if $M$ has a nonzero column, then $N=0$; and if $N$ has a nonzero row, then $M=0$.






share|cite|improve this answer























    Your Answer





    StackExchange.ifUsing("editor", function () {
    return StackExchange.using("mathjaxEditing", function () {
    StackExchange.MarkdownEditor.creationCallbacks.add(function (editor, postfix) {
    StackExchange.mathjaxEditing.prepareWmdForMathJax(editor, postfix, [["$", "$"], ["\\(","\\)"]]);
    });
    });
    }, "mathjax-editing");

    StackExchange.ready(function() {
    var channelOptions = {
    tags: "".split(" "),
    id: "69"
    };
    initTagRenderer("".split(" "), "".split(" "), channelOptions);

    StackExchange.using("externalEditor", function() {
    // Have to fire editor after snippets, if snippets enabled
    if (StackExchange.settings.snippets.snippetsEnabled) {
    StackExchange.using("snippets", function() {
    createEditor();
    });
    }
    else {
    createEditor();
    }
    });

    function createEditor() {
    StackExchange.prepareEditor({
    heartbeatType: 'answer',
    convertImagesToLinks: true,
    noModals: true,
    showLowRepImageUploadWarning: true,
    reputationToPostImages: 10,
    bindNavPrevention: true,
    postfix: "",
    imageUploader: {
    brandingHtml: "Powered by u003ca class="icon-imgur-white" href="https://imgur.com/"u003eu003c/au003e",
    contentPolicyHtml: "User contributions licensed under u003ca href="https://creativecommons.org/licenses/by-sa/3.0/"u003ecc by-sa 3.0 with attribution requiredu003c/au003e u003ca href="https://stackoverflow.com/legal/content-policy"u003e(content policy)u003c/au003e",
    allowUrls: true
    },
    noCode: true, onDemand: true,
    discardSelector: ".discard-answer"
    ,immediatelyShowMarkdownHelp:true
    });


    }
    });














    draft saved

    draft discarded


















    StackExchange.ready(
    function () {
    StackExchange.openid.initPostLogin('.new-post-login', 'https%3a%2f%2fmath.stackexchange.com%2fquestions%2f3008710%2fis-it-possible-to-find-matrices-m-n-such-that-mxn-has-specified-structure-f%23new-answer', 'question_page');
    }
    );

    Post as a guest















    Required, but never shown

























    1 Answer
    1






    active

    oldest

    votes








    1 Answer
    1






    active

    oldest

    votes









    active

    oldest

    votes






    active

    oldest

    votes








    up vote
    1
    down vote



    accepted










    No, it's not possible unless you accept that the range is ${0}$. Consider first the case $1,1$. We have
    $$
    begin{bmatrix} M_{11}\ M_{21}end{bmatrix}begin{bmatrix} N_{11}&N_{12}end{bmatrix}=begin{bmatrix} 0&a\a&0end{bmatrix}.
    $$

    Then $M_{11}N_{11}=0$. If $M_{11}ne0$, then $N_{11}=0$. But then $a=M_{21}N_{11}=0$. From the $1,2$ entry, $0=a=M_{11}N_{12}$, $N_{12}=0$ and $N=0$. When $N_{11}=0$, we obtain similarly that $M=0$ (just take transpose and apply the argument).



    Now in general, if $E_{kj}$ is the matrix with $1$ in the $k,j$ entry and zeroes elsewhere,
    $$
    ME_{kj}N=M_kN_j,
    $$

    where $M_k$ is the $k^{rm th}$ column of $M$ and $N_j$ with the $j^{rm th}$ row of $j$. By the previous case, one of them has to be zero. So if $M$ has a nonzero column, then $N=0$; and if $N$ has a nonzero row, then $M=0$.






    share|cite|improve this answer



























      up vote
      1
      down vote



      accepted










      No, it's not possible unless you accept that the range is ${0}$. Consider first the case $1,1$. We have
      $$
      begin{bmatrix} M_{11}\ M_{21}end{bmatrix}begin{bmatrix} N_{11}&N_{12}end{bmatrix}=begin{bmatrix} 0&a\a&0end{bmatrix}.
      $$

      Then $M_{11}N_{11}=0$. If $M_{11}ne0$, then $N_{11}=0$. But then $a=M_{21}N_{11}=0$. From the $1,2$ entry, $0=a=M_{11}N_{12}$, $N_{12}=0$ and $N=0$. When $N_{11}=0$, we obtain similarly that $M=0$ (just take transpose and apply the argument).



      Now in general, if $E_{kj}$ is the matrix with $1$ in the $k,j$ entry and zeroes elsewhere,
      $$
      ME_{kj}N=M_kN_j,
      $$

      where $M_k$ is the $k^{rm th}$ column of $M$ and $N_j$ with the $j^{rm th}$ row of $j$. By the previous case, one of them has to be zero. So if $M$ has a nonzero column, then $N=0$; and if $N$ has a nonzero row, then $M=0$.






      share|cite|improve this answer

























        up vote
        1
        down vote



        accepted







        up vote
        1
        down vote



        accepted






        No, it's not possible unless you accept that the range is ${0}$. Consider first the case $1,1$. We have
        $$
        begin{bmatrix} M_{11}\ M_{21}end{bmatrix}begin{bmatrix} N_{11}&N_{12}end{bmatrix}=begin{bmatrix} 0&a\a&0end{bmatrix}.
        $$

        Then $M_{11}N_{11}=0$. If $M_{11}ne0$, then $N_{11}=0$. But then $a=M_{21}N_{11}=0$. From the $1,2$ entry, $0=a=M_{11}N_{12}$, $N_{12}=0$ and $N=0$. When $N_{11}=0$, we obtain similarly that $M=0$ (just take transpose and apply the argument).



        Now in general, if $E_{kj}$ is the matrix with $1$ in the $k,j$ entry and zeroes elsewhere,
        $$
        ME_{kj}N=M_kN_j,
        $$

        where $M_k$ is the $k^{rm th}$ column of $M$ and $N_j$ with the $j^{rm th}$ row of $j$. By the previous case, one of them has to be zero. So if $M$ has a nonzero column, then $N=0$; and if $N$ has a nonzero row, then $M=0$.






        share|cite|improve this answer














        No, it's not possible unless you accept that the range is ${0}$. Consider first the case $1,1$. We have
        $$
        begin{bmatrix} M_{11}\ M_{21}end{bmatrix}begin{bmatrix} N_{11}&N_{12}end{bmatrix}=begin{bmatrix} 0&a\a&0end{bmatrix}.
        $$

        Then $M_{11}N_{11}=0$. If $M_{11}ne0$, then $N_{11}=0$. But then $a=M_{21}N_{11}=0$. From the $1,2$ entry, $0=a=M_{11}N_{12}$, $N_{12}=0$ and $N=0$. When $N_{11}=0$, we obtain similarly that $M=0$ (just take transpose and apply the argument).



        Now in general, if $E_{kj}$ is the matrix with $1$ in the $k,j$ entry and zeroes elsewhere,
        $$
        ME_{kj}N=M_kN_j,
        $$

        where $M_k$ is the $k^{rm th}$ column of $M$ and $N_j$ with the $j^{rm th}$ row of $j$. By the previous case, one of them has to be zero. So if $M$ has a nonzero column, then $N=0$; and if $N$ has a nonzero row, then $M=0$.







        share|cite|improve this answer














        share|cite|improve this answer



        share|cite|improve this answer








        edited Nov 26 at 4:21

























        answered Nov 22 at 4:45









        Martin Argerami

        122k1176173




        122k1176173






























            draft saved

            draft discarded




















































            Thanks for contributing an answer to Mathematics Stack Exchange!


            • Please be sure to answer the question. Provide details and share your research!

            But avoid



            • Asking for help, clarification, or responding to other answers.

            • Making statements based on opinion; back them up with references or personal experience.


            Use MathJax to format equations. MathJax reference.


            To learn more, see our tips on writing great answers.





            Some of your past answers have not been well-received, and you're in danger of being blocked from answering.


            Please pay close attention to the following guidance:


            • Please be sure to answer the question. Provide details and share your research!

            But avoid



            • Asking for help, clarification, or responding to other answers.

            • Making statements based on opinion; back them up with references or personal experience.


            To learn more, see our tips on writing great answers.




            draft saved


            draft discarded














            StackExchange.ready(
            function () {
            StackExchange.openid.initPostLogin('.new-post-login', 'https%3a%2f%2fmath.stackexchange.com%2fquestions%2f3008710%2fis-it-possible-to-find-matrices-m-n-such-that-mxn-has-specified-structure-f%23new-answer', 'question_page');
            }
            );

            Post as a guest















            Required, but never shown





















































            Required, but never shown














            Required, but never shown












            Required, but never shown







            Required, but never shown

































            Required, but never shown














            Required, but never shown












            Required, but never shown







            Required, but never shown







            Popular posts from this blog

            Bundesstraße 106

            Le Mesnil-Réaume

            Ida-Boy-Ed-Garten